Please confirm topic selection

Are you sure you want to trigger topic in your Anconeus AI algorithm?

Please confirm action

You are done for today with this topic.

Would you like to start learning session with this topic items scheduled for future?

Review Question - QID 108728

In scope icon M 7 D
QID 108728 (Type "108728" in App Search)
A 43-year-old woman presents to her primary care provider for a follow-up appointment. During her annual visit 3 months ago, she had no complaints but was noted to have lost 8 pounds since her appointment the previous year. She denied any changes to her exercise regimen but endorsed trying to limit her soda consumption. She also endorsed having “hot flashes” despite continuing to have regular menses. At the time, she was found to have the physical exam finding shown in Figure A. Laboratory studies showed a suppressed thyroid-stimulating hormone (TSH) level and elevated free T4 level. Since then, she has been taking 2 medications for her condition. She reports experiencing less anxiety and her weight has stabilized. She has no other medical problems and takes no other medications. She does not smoke, drink alcohol, or use drugs. Her temperature is 97.9°F (36.6°C), blood pressure is 133/84 mmHg, pulse is 81/min, and respirations are 12/min. Physical exam reveals the same finding shown in Figure A. Routine laboratory testing is performed which reveals the following:

Leukocyte count: 0.2 x 10^3/mm^3
Segmented neutrophils: 40%
Bands: 1%
Eosinophils: 1%
Basophils: 0.6%
Lymphocytes: 46.4%
Monocytes: 11%

Hemoglobin: 13.1 g/dL
Hematocrit: 44%
Mean corpuscular volume: 92 µm^3
Platelet count: 312,000/mm^3

Which of the following is the most appropriate next step in management?
  • A
  • A